A Very Illegal Position












31















It's been a while since I've done a chess puzzle, so here's one!



This position is illegal in quite a lot of ways:



enter image description here



It's illegal in at least 10 ways. The first one to find all of the ways this position is illegal (it's 10 or more, I know exactly how many) gets the check and +1 from me.



Good luck and happy puzzling!



Updated: I am considering, for the sake of this question, a way that this position is illegal to be any part of the position that could not have occurred from the starting position, even if it requires other illegal pieces to be that way.










share|improve this question




















  • 1





    It might help if you had a more specific definition of what a "way the position is illegal" consists of. I think I'm using a different definition, which is why I can only find 8.

    – isaacg
    Jan 3 at 23:01











  • Obviously there's no way for the queen to still be on the board.

    – Steve
    Jan 4 at 17:57
















31















It's been a while since I've done a chess puzzle, so here's one!



This position is illegal in quite a lot of ways:



enter image description here



It's illegal in at least 10 ways. The first one to find all of the ways this position is illegal (it's 10 or more, I know exactly how many) gets the check and +1 from me.



Good luck and happy puzzling!



Updated: I am considering, for the sake of this question, a way that this position is illegal to be any part of the position that could not have occurred from the starting position, even if it requires other illegal pieces to be that way.










share|improve this question




















  • 1





    It might help if you had a more specific definition of what a "way the position is illegal" consists of. I think I'm using a different definition, which is why I can only find 8.

    – isaacg
    Jan 3 at 23:01











  • Obviously there's no way for the queen to still be on the board.

    – Steve
    Jan 4 at 17:57














31












31








31


5






It's been a while since I've done a chess puzzle, so here's one!



This position is illegal in quite a lot of ways:



enter image description here



It's illegal in at least 10 ways. The first one to find all of the ways this position is illegal (it's 10 or more, I know exactly how many) gets the check and +1 from me.



Good luck and happy puzzling!



Updated: I am considering, for the sake of this question, a way that this position is illegal to be any part of the position that could not have occurred from the starting position, even if it requires other illegal pieces to be that way.










share|improve this question
















It's been a while since I've done a chess puzzle, so here's one!



This position is illegal in quite a lot of ways:



enter image description here



It's illegal in at least 10 ways. The first one to find all of the ways this position is illegal (it's 10 or more, I know exactly how many) gets the check and +1 from me.



Good luck and happy puzzling!



Updated: I am considering, for the sake of this question, a way that this position is illegal to be any part of the position that could not have occurred from the starting position, even if it requires other illegal pieces to be that way.







chess






share|improve this question















share|improve this question













share|improve this question




share|improve this question








edited Jan 4 at 11:15







Excited Raichu

















asked Jan 3 at 14:58









Excited RaichuExcited Raichu

6,22821065




6,22821065








  • 1





    It might help if you had a more specific definition of what a "way the position is illegal" consists of. I think I'm using a different definition, which is why I can only find 8.

    – isaacg
    Jan 3 at 23:01











  • Obviously there's no way for the queen to still be on the board.

    – Steve
    Jan 4 at 17:57














  • 1





    It might help if you had a more specific definition of what a "way the position is illegal" consists of. I think I'm using a different definition, which is why I can only find 8.

    – isaacg
    Jan 3 at 23:01











  • Obviously there's no way for the queen to still be on the board.

    – Steve
    Jan 4 at 17:57








1




1





It might help if you had a more specific definition of what a "way the position is illegal" consists of. I think I'm using a different definition, which is why I can only find 8.

– isaacg
Jan 3 at 23:01





It might help if you had a more specific definition of what a "way the position is illegal" consists of. I think I'm using a different definition, which is why I can only find 8.

– isaacg
Jan 3 at 23:01













Obviously there's no way for the queen to still be on the board.

– Steve
Jan 4 at 17:57





Obviously there's no way for the queen to still be on the board.

– Steve
Jan 4 at 17:57










10 Answers
10






active

oldest

votes


















16














1.




Both kings are in check.




2.




The black king is in check by both the Rc3 and Bg4. Double-check is possible in certain cases, by either a discovered check or an en-passant capture, but neither can have happened here.




3.




The white bishop on b1 can only arrive there via a2 and c2, and there are white pawns on a2 and c2 which can't have moved.




4.




Black has nine pawns.




5.




White has two light-squared bishops; one must have been promoted but White has still eight pawns.




6.




White's pawns seem to have captured three times (b-pawn to d5, d-pawn to e3), but Black's only missing piece is a single knight. Also, the f7 pawn can only get there via captures (unless the black f6 pawn somehow dropped out of thin air, see 4.)




7.




There's no way for the black rook which started on h8 to escape (to either b6 or a1).






I'm not sure I can find more; for example,




it will be hard for the Ra1 to reach that square, but once you assume the Bb1 materialized out of thin air (see 3.), it's not a problem anymore.




Also,




the black pawns are 7 columns 'away' from their home squares, but if you assume the g3 has been dropped there, only 4 captures are required to account for the a- and b-pawns, and there are four white pieces missing (two knights, a rook and the queen).




A pitfall:




the white king can be put in check this way; Black's last move could have been Nh4+ (discovered check)




Note that




some of these irregularities could happen during a game of bughouse.







share|improve this answer


























  • rook h8 could have been captured by a white knight, though

    – Bass
    Jan 3 at 15:21






  • 3





    True, but that would require another promoted black pawn.

    – Glorfindel
    Jan 3 at 15:22











  • Note that in a game of antichess bughouse, this is probably a valid position :)

    – Hosch250
    Jan 4 at 14:08











  • On #2, it might be worth elaborating. Since, in real games, it's possible to have a double check when one check is revealed by the second checking piece. But in this particular case, there's no way for the rook or the bishop to have been blocking the other before the most recent move.

    – Shufflepants
    Jan 4 at 16:03











  • @WW That assumes the last move was black's.

    – Shufflepants
    Jan 4 at 18:29



















7














Here's my 10 (I didn't look at anyone else's answers, honest)



1.




Nine black pawns.




2.




Black's h8 rook could not have left rank 8, yet black has two rooks, entailing a promotion from a tenth black pawn!




3.




Black pawns have made at least five captures, e.g. cxbxa3 and dxexfxg3, but White is missing only four units (queen, rook and two knights)




4.




Black's check with the bishop on f3 had to be a capture -- yet another capture by Black, and this capture, too, is invalid for the same reason as given in 3.




5.




White pawns have made at least three captures towards the kingside, e.g. bxcxd5 and dxe3. Moreover, two captures fxexf or fxgxf are needed to put a white pawn on f7. However, even if we remove a black pawn, black is missing only one unit (a knight -- correcting an oversight noted by Separatrix).




6.




White's check with the bishop on g4 had to be a capture -- yet another capture by White, and this capture, too, is invalid for the same reason as given in 5.




7.




White's bishop on g4 and rook on c3 are giving an impossible double check.




8.




4. and 6. mean both kings are in check at the same time.




9.




White bishop on b1, though it wasn't born there and couldn't have moved there.




10.




White has two bishops on white squares but has eight pawns and thus has no promoted pieces.







share|improve this answer


























  • #4 is incorrect: Black last move could be Ng2-h4.

    – Evargalo
    Jan 4 at 7:11











  • @Evargalo Oh yes, fair point.

    – Rosie F
    Jan 4 at 7:45











  • #5 Black has both rooks, missing a knight.

    – Separatrix
    Jan 4 at 11:53



















6














I'm not sure how you're counting, but here's my answer. Possibly an incomplete answer.



1.




Both kings are in check.




2.




The black king is in check from two pieces.




3.




There are 9 black pawns. Removing the one at g3 makes the pawns at a3 and b2 valid via capturing white officers.




4.




There are not enough captured black pieces for the white pawns to be in those positions. The f7 pawn in particular is in a suspicious location, though this pawn configuration is possible if there were enough black officers to capture.




5.




The white bishop on b1 is in an impossible location.




6.




The white bishops are both on white squares, but all of the pawns are present.




7.




The black rook at a1 is in an impossible location... Sort of. It would be possible via promotion, but there already too many black pawns for that to be possible.




8.




There should be a black rook in the upper-right corner because there's no way it could get out with the bishop at f8.




That's all I see. I might be lumping together multiple items by your count into one.






share|improve this answer































    5















    1. bishop in g4 in check

    2. rook in c3 in check

    3. bishop in f3 in check

    4. rook in a1 cant get there

    5. 9 black pawns

    6. pawn in a3 cant get there

    7. pawn in g3 cant get there

    8. pawn in f7 cant get there

    9. 2 white bishop on white squares

    10. pawn in e3 cant get there

    11. bishop in b1 cant get there







    share|improve this answer





















    • 1





      Your #4 and #12 are the same. The A3 pawn could have come from the C column by capturing twice, same goes for G3 from the D column, although since black is only missing 2 pieces, one of these must be illegal.

      – Nuclear Wang
      Jan 3 at 15:36



















    4














    1.




    Black has 9 pawns.




    2.




    Both kings are in check.




    3.




    The black king is in double check. This can only happen if one of the checking pieces shielded the opponent king from the other prior to the check. This is not possible with the took on c3 and the bishop on h4.




    4.




    The white bishop on b1 could never have reached this field with the white pawns on a2, c2.




    5.




    White has two bishops on white fields. This can only happen after a pawn has been promoted. White however still has their complete set of 8 pawns.




    6.




    With white pawns on the e and g files and a black pawn on f6, the white pawn on f7 can reach this field only with at least 2 captures. However, black has lost only 1 piece and all pawns (in fact even one surplus pawn, he. item 1), thus never had a promoted piece.




    7.




    White's pawn structure implies at least 5 captures (bxc, cxd; dxe; fxe, exf) but black has lost a single piece only and no promotion so far (cf. item 6)




    8.




    With black pawns on e7,f6,g7,h7 and a black bishop on f8, the black rook initially on h8 could only have reached the field g8.




    9.




    The black king is in check so white moved last. Since the white bishop on h4 can only move along the diagonal to c8 the black king must have been in check before the move; or black put itself into check by illegally moving a pinned piece; or white moved a different piece to open the diagonal, but white does not have a piece that could have moved this way [not sure if this counts though as the checking situation is impossible in the first place, cf. items 2 & 3]




    10.




    Black is in an 'impossible' double check and puts white in check (cf. items 2 & 3). That can only happen if black doesn't move out of a check, thus playing illegally.







    share|improve this answer

































      3














      I only managed to find 10 so far, here it is:




      1. White bishop in B1. It's not possible in a normal game.

      2. Two white bishops in the white square, while still having 8 pawns.

      3. Black Rook in A1. It's not possible in a normal game.

      4. Black Rook going out from H8 while Bishop hasn't moved from F8.

      5. White Rook missing from A1. Stolen and replaced by black rook? :P

      6. Black pawn totaling 9. It was supposed to be only 8 in a game.

      7. White king under check, while black king also in check.

      8. Black king under two different check, from Rook in C3 and Bishop on G4

      9. White pawn structure. It just simply weird to have a pawn on F7, D5 while also having a pawn on E3, while black only missing a knight.

      10. Both kings is under check, is simply impossible.







      share|improve this answer
























      • Black rook in A1 is possible (in isolation) as a promoted pawn having captured the White rook normally there. #7 and #10 are duplicates.

        – Separatrix
        Jan 4 at 12:06








      • 1





        @Separatrix: Black has 9 pawns (in itself illegal), however, so there has not been a promotion. Or, if there was, black has been playing with at least 10 pawns at some moment in time, adding another illegal pawn (unless surplus pawns count as a single rule violation w/o regard to their number).

        – collapsar
        Jan 4 at 17:25



















      3














      Definitely illegal



      1.




      Black has 9 pawns




      2.




      Rook a1 could be legal in isolation as a promoted pawn, but in light of #1, I'm declaring it illegal




      3.




      Bishop b1 cannot get there




      4.




      Both white bishops are on white, white has 8 pawns, one bishop is illegal




      5.




      Both kings are in check




      6.




      Black king is in illegal double check, could not be discovered by legal movement of either offending piece




      7.




      Black rook starting h8 could not have legally escaped, due to number of remaining black pawns, absence of rook considered illegal




      8.




      White pawns are at least 3 places off home columns, black still has 16 pieces, though missing a knight, at least two illegal places off home




      9.




      Pawn f7 is illegal, cannot have captured the required 2 pieces to return to home column behind black pawn as black not missing enough pieces




      10.




      Even assuming pawn g3 to be the interloper, black pawns are a minimum of 4 places off home column, white is missing 4 pieces, but rook a1 could only have been captured by a knight or queen, at least one illegal place off home




      Not illegal but might look it



      1.




      Bishop f3 check could be discovered by movement of knight h4 from g2, otherwise it's really hard to get into that position





      1. This ties into illegal #10



      Assuming bishop b1 to have been placed there at the start of the game, white rook starting a1 could not have legally escaped, but could have been captured by a knight or queen. Black bishop could not have been used.







      share|improve this answer

































        2
















        • Bb1, no legal way to reach there



        • -


        - Ra1, no legal way to reach there, due to Bb1






        • Kh1, in check, other King in check



        • -


        Kc8, in double check, other King in check






        • Kc8, in an impossible double check, no legal way to give that double check



        • -


        f7, no legal way to reach there






        • Bf8, no legal way to reach there



        • -


        2 white bishops, with 8 pawns. Does this count as two illegals?







        share|improve this answer

































          1














          1.




          Both kings are in check.




          2.




          Black is in check by 2 pieces, which, as arranged, could not have co-participated in a discovered check.




          3.




          Black has 9 pawns.




          4.




          White's bishop could not reach B1 without a pawn moving backwards.




          5.




          Black's rook could not have reached A1.




          6.




          Both White bishops are on the same color.




          7.




          White pawn at F7 cannot have passed through Black pawn at F6, nor could it have reached that position through capture as Black is missing at most one piece.




          8.




          White pawn at position E3 is impossible without capturing more material than Black lost.




          9.




          White pawn from file B is unaccounted for. With Black down at most one piece, it should be in file A, B, or C.




          10.




          Black rook from H8 could not have left starting position (other than G8 and it's not there either).




          A.




          It's tempting to say that positioning of Black's pawns is illegal, but I've already taken credit for one illegal pawn. If A3 is that illegal pawn, then it could have started at, say, A6. Then B2 could be reached by capture of 1 piece, and G3 could be reached by capture of 3 pieces, and White is down in material by 4 pieces, so it's not impossible. It depends on whether Black started out with 16 pieces within its two starting ranks (substituting a pawn for a knight), or if the 9th pawn started the game outside of those two ranks.







          share|improve this answer































            0














            1.




            Black has a 9th pawn.




            2.




            Both White and Black are in check simultaneously.




            3.




            Black is in double check with the c3 rook and g4 bishop.




            4.




            White's two bishops are on the same color of square. White still has 8 pawns, so one bishop could not have come from promotion. Either a bishop started on the wrong square, or White started with additional pawns and/or bishops.




            5.




            White's bishop on b1, if it started on f1, could not have got to b1 when the diagonal pawns are still in place.




            6.




            Black's rook on a1 could not have got there behind the b1 bishop, if the bishop started there.




            7.




            Black's rook that would have started on h8 could not have gotten out of the g8-h8 box.




            8.




            White's pawns on d5 and e3 and f7 require that at least 5 diagonal movements - captures of Black's pieces - have taken place, but Black is down at most 1 piece (a knight), or started with several additional pieces.




            Fun! But I count only eight.






            share|improve this answer

























              Your Answer





              StackExchange.ifUsing("editor", function () {
              return StackExchange.using("mathjaxEditing", function () {
              StackExchange.MarkdownEditor.creationCallbacks.add(function (editor, postfix) {
              StackExchange.mathjaxEditing.prepareWmdForMathJax(editor, postfix, [["$", "$"], ["\\(","\\)"]]);
              });
              });
              }, "mathjax-editing");

              StackExchange.ready(function() {
              var channelOptions = {
              tags: "".split(" "),
              id: "559"
              };
              initTagRenderer("".split(" "), "".split(" "), channelOptions);

              StackExchange.using("externalEditor", function() {
              // Have to fire editor after snippets, if snippets enabled
              if (StackExchange.settings.snippets.snippetsEnabled) {
              StackExchange.using("snippets", function() {
              createEditor();
              });
              }
              else {
              createEditor();
              }
              });

              function createEditor() {
              StackExchange.prepareEditor({
              heartbeatType: 'answer',
              autoActivateHeartbeat: false,
              convertImagesToLinks: false,
              noModals: true,
              showLowRepImageUploadWarning: true,
              reputationToPostImages: null,
              bindNavPrevention: true,
              postfix: "",
              imageUploader: {
              brandingHtml: "Powered by u003ca class="icon-imgur-white" href="https://imgur.com/"u003eu003c/au003e",
              contentPolicyHtml: "User contributions licensed under u003ca href="https://creativecommons.org/licenses/by-sa/3.0/"u003ecc by-sa 3.0 with attribution requiredu003c/au003e u003ca href="https://stackoverflow.com/legal/content-policy"u003e(content policy)u003c/au003e",
              allowUrls: true
              },
              noCode: true, onDemand: true,
              discardSelector: ".discard-answer"
              ,immediatelyShowMarkdownHelp:true
              });


              }
              });














              draft saved

              draft discarded


















              StackExchange.ready(
              function () {
              StackExchange.openid.initPostLogin('.new-post-login', 'https%3a%2f%2fpuzzling.stackexchange.com%2fquestions%2f78081%2fa-very-illegal-position%23new-answer', 'question_page');
              }
              );

              Post as a guest















              Required, but never shown

























              10 Answers
              10






              active

              oldest

              votes








              10 Answers
              10






              active

              oldest

              votes









              active

              oldest

              votes






              active

              oldest

              votes









              16














              1.




              Both kings are in check.




              2.




              The black king is in check by both the Rc3 and Bg4. Double-check is possible in certain cases, by either a discovered check or an en-passant capture, but neither can have happened here.




              3.




              The white bishop on b1 can only arrive there via a2 and c2, and there are white pawns on a2 and c2 which can't have moved.




              4.




              Black has nine pawns.




              5.




              White has two light-squared bishops; one must have been promoted but White has still eight pawns.




              6.




              White's pawns seem to have captured three times (b-pawn to d5, d-pawn to e3), but Black's only missing piece is a single knight. Also, the f7 pawn can only get there via captures (unless the black f6 pawn somehow dropped out of thin air, see 4.)




              7.




              There's no way for the black rook which started on h8 to escape (to either b6 or a1).






              I'm not sure I can find more; for example,




              it will be hard for the Ra1 to reach that square, but once you assume the Bb1 materialized out of thin air (see 3.), it's not a problem anymore.




              Also,




              the black pawns are 7 columns 'away' from their home squares, but if you assume the g3 has been dropped there, only 4 captures are required to account for the a- and b-pawns, and there are four white pieces missing (two knights, a rook and the queen).




              A pitfall:




              the white king can be put in check this way; Black's last move could have been Nh4+ (discovered check)




              Note that




              some of these irregularities could happen during a game of bughouse.







              share|improve this answer


























              • rook h8 could have been captured by a white knight, though

                – Bass
                Jan 3 at 15:21






              • 3





                True, but that would require another promoted black pawn.

                – Glorfindel
                Jan 3 at 15:22











              • Note that in a game of antichess bughouse, this is probably a valid position :)

                – Hosch250
                Jan 4 at 14:08











              • On #2, it might be worth elaborating. Since, in real games, it's possible to have a double check when one check is revealed by the second checking piece. But in this particular case, there's no way for the rook or the bishop to have been blocking the other before the most recent move.

                – Shufflepants
                Jan 4 at 16:03











              • @WW That assumes the last move was black's.

                – Shufflepants
                Jan 4 at 18:29
















              16














              1.




              Both kings are in check.




              2.




              The black king is in check by both the Rc3 and Bg4. Double-check is possible in certain cases, by either a discovered check or an en-passant capture, but neither can have happened here.




              3.




              The white bishop on b1 can only arrive there via a2 and c2, and there are white pawns on a2 and c2 which can't have moved.




              4.




              Black has nine pawns.




              5.




              White has two light-squared bishops; one must have been promoted but White has still eight pawns.




              6.




              White's pawns seem to have captured three times (b-pawn to d5, d-pawn to e3), but Black's only missing piece is a single knight. Also, the f7 pawn can only get there via captures (unless the black f6 pawn somehow dropped out of thin air, see 4.)




              7.




              There's no way for the black rook which started on h8 to escape (to either b6 or a1).






              I'm not sure I can find more; for example,




              it will be hard for the Ra1 to reach that square, but once you assume the Bb1 materialized out of thin air (see 3.), it's not a problem anymore.




              Also,




              the black pawns are 7 columns 'away' from their home squares, but if you assume the g3 has been dropped there, only 4 captures are required to account for the a- and b-pawns, and there are four white pieces missing (two knights, a rook and the queen).




              A pitfall:




              the white king can be put in check this way; Black's last move could have been Nh4+ (discovered check)




              Note that




              some of these irregularities could happen during a game of bughouse.







              share|improve this answer


























              • rook h8 could have been captured by a white knight, though

                – Bass
                Jan 3 at 15:21






              • 3





                True, but that would require another promoted black pawn.

                – Glorfindel
                Jan 3 at 15:22











              • Note that in a game of antichess bughouse, this is probably a valid position :)

                – Hosch250
                Jan 4 at 14:08











              • On #2, it might be worth elaborating. Since, in real games, it's possible to have a double check when one check is revealed by the second checking piece. But in this particular case, there's no way for the rook or the bishop to have been blocking the other before the most recent move.

                – Shufflepants
                Jan 4 at 16:03











              • @WW That assumes the last move was black's.

                – Shufflepants
                Jan 4 at 18:29














              16












              16








              16







              1.




              Both kings are in check.




              2.




              The black king is in check by both the Rc3 and Bg4. Double-check is possible in certain cases, by either a discovered check or an en-passant capture, but neither can have happened here.




              3.




              The white bishop on b1 can only arrive there via a2 and c2, and there are white pawns on a2 and c2 which can't have moved.




              4.




              Black has nine pawns.




              5.




              White has two light-squared bishops; one must have been promoted but White has still eight pawns.




              6.




              White's pawns seem to have captured three times (b-pawn to d5, d-pawn to e3), but Black's only missing piece is a single knight. Also, the f7 pawn can only get there via captures (unless the black f6 pawn somehow dropped out of thin air, see 4.)




              7.




              There's no way for the black rook which started on h8 to escape (to either b6 or a1).






              I'm not sure I can find more; for example,




              it will be hard for the Ra1 to reach that square, but once you assume the Bb1 materialized out of thin air (see 3.), it's not a problem anymore.




              Also,




              the black pawns are 7 columns 'away' from their home squares, but if you assume the g3 has been dropped there, only 4 captures are required to account for the a- and b-pawns, and there are four white pieces missing (two knights, a rook and the queen).




              A pitfall:




              the white king can be put in check this way; Black's last move could have been Nh4+ (discovered check)




              Note that




              some of these irregularities could happen during a game of bughouse.







              share|improve this answer















              1.




              Both kings are in check.




              2.




              The black king is in check by both the Rc3 and Bg4. Double-check is possible in certain cases, by either a discovered check or an en-passant capture, but neither can have happened here.




              3.




              The white bishop on b1 can only arrive there via a2 and c2, and there are white pawns on a2 and c2 which can't have moved.




              4.




              Black has nine pawns.




              5.




              White has two light-squared bishops; one must have been promoted but White has still eight pawns.




              6.




              White's pawns seem to have captured three times (b-pawn to d5, d-pawn to e3), but Black's only missing piece is a single knight. Also, the f7 pawn can only get there via captures (unless the black f6 pawn somehow dropped out of thin air, see 4.)




              7.




              There's no way for the black rook which started on h8 to escape (to either b6 or a1).






              I'm not sure I can find more; for example,




              it will be hard for the Ra1 to reach that square, but once you assume the Bb1 materialized out of thin air (see 3.), it's not a problem anymore.




              Also,




              the black pawns are 7 columns 'away' from their home squares, but if you assume the g3 has been dropped there, only 4 captures are required to account for the a- and b-pawns, and there are four white pieces missing (two knights, a rook and the queen).




              A pitfall:




              the white king can be put in check this way; Black's last move could have been Nh4+ (discovered check)




              Note that




              some of these irregularities could happen during a game of bughouse.








              share|improve this answer














              share|improve this answer



              share|improve this answer








              edited Jan 4 at 16:05

























              answered Jan 3 at 15:02









              GlorfindelGlorfindel

              13.6k34983




              13.6k34983













              • rook h8 could have been captured by a white knight, though

                – Bass
                Jan 3 at 15:21






              • 3





                True, but that would require another promoted black pawn.

                – Glorfindel
                Jan 3 at 15:22











              • Note that in a game of antichess bughouse, this is probably a valid position :)

                – Hosch250
                Jan 4 at 14:08











              • On #2, it might be worth elaborating. Since, in real games, it's possible to have a double check when one check is revealed by the second checking piece. But in this particular case, there's no way for the rook or the bishop to have been blocking the other before the most recent move.

                – Shufflepants
                Jan 4 at 16:03











              • @WW That assumes the last move was black's.

                – Shufflepants
                Jan 4 at 18:29



















              • rook h8 could have been captured by a white knight, though

                – Bass
                Jan 3 at 15:21






              • 3





                True, but that would require another promoted black pawn.

                – Glorfindel
                Jan 3 at 15:22











              • Note that in a game of antichess bughouse, this is probably a valid position :)

                – Hosch250
                Jan 4 at 14:08











              • On #2, it might be worth elaborating. Since, in real games, it's possible to have a double check when one check is revealed by the second checking piece. But in this particular case, there's no way for the rook or the bishop to have been blocking the other before the most recent move.

                – Shufflepants
                Jan 4 at 16:03











              • @WW That assumes the last move was black's.

                – Shufflepants
                Jan 4 at 18:29

















              rook h8 could have been captured by a white knight, though

              – Bass
              Jan 3 at 15:21





              rook h8 could have been captured by a white knight, though

              – Bass
              Jan 3 at 15:21




              3




              3





              True, but that would require another promoted black pawn.

              – Glorfindel
              Jan 3 at 15:22





              True, but that would require another promoted black pawn.

              – Glorfindel
              Jan 3 at 15:22













              Note that in a game of antichess bughouse, this is probably a valid position :)

              – Hosch250
              Jan 4 at 14:08





              Note that in a game of antichess bughouse, this is probably a valid position :)

              – Hosch250
              Jan 4 at 14:08













              On #2, it might be worth elaborating. Since, in real games, it's possible to have a double check when one check is revealed by the second checking piece. But in this particular case, there's no way for the rook or the bishop to have been blocking the other before the most recent move.

              – Shufflepants
              Jan 4 at 16:03





              On #2, it might be worth elaborating. Since, in real games, it's possible to have a double check when one check is revealed by the second checking piece. But in this particular case, there's no way for the rook or the bishop to have been blocking the other before the most recent move.

              – Shufflepants
              Jan 4 at 16:03













              @WW That assumes the last move was black's.

              – Shufflepants
              Jan 4 at 18:29





              @WW That assumes the last move was black's.

              – Shufflepants
              Jan 4 at 18:29











              7














              Here's my 10 (I didn't look at anyone else's answers, honest)



              1.




              Nine black pawns.




              2.




              Black's h8 rook could not have left rank 8, yet black has two rooks, entailing a promotion from a tenth black pawn!




              3.




              Black pawns have made at least five captures, e.g. cxbxa3 and dxexfxg3, but White is missing only four units (queen, rook and two knights)




              4.




              Black's check with the bishop on f3 had to be a capture -- yet another capture by Black, and this capture, too, is invalid for the same reason as given in 3.




              5.




              White pawns have made at least three captures towards the kingside, e.g. bxcxd5 and dxe3. Moreover, two captures fxexf or fxgxf are needed to put a white pawn on f7. However, even if we remove a black pawn, black is missing only one unit (a knight -- correcting an oversight noted by Separatrix).




              6.




              White's check with the bishop on g4 had to be a capture -- yet another capture by White, and this capture, too, is invalid for the same reason as given in 5.




              7.




              White's bishop on g4 and rook on c3 are giving an impossible double check.




              8.




              4. and 6. mean both kings are in check at the same time.




              9.




              White bishop on b1, though it wasn't born there and couldn't have moved there.




              10.




              White has two bishops on white squares but has eight pawns and thus has no promoted pieces.







              share|improve this answer


























              • #4 is incorrect: Black last move could be Ng2-h4.

                – Evargalo
                Jan 4 at 7:11











              • @Evargalo Oh yes, fair point.

                – Rosie F
                Jan 4 at 7:45











              • #5 Black has both rooks, missing a knight.

                – Separatrix
                Jan 4 at 11:53
















              7














              Here's my 10 (I didn't look at anyone else's answers, honest)



              1.




              Nine black pawns.




              2.




              Black's h8 rook could not have left rank 8, yet black has two rooks, entailing a promotion from a tenth black pawn!




              3.




              Black pawns have made at least five captures, e.g. cxbxa3 and dxexfxg3, but White is missing only four units (queen, rook and two knights)




              4.




              Black's check with the bishop on f3 had to be a capture -- yet another capture by Black, and this capture, too, is invalid for the same reason as given in 3.




              5.




              White pawns have made at least three captures towards the kingside, e.g. bxcxd5 and dxe3. Moreover, two captures fxexf or fxgxf are needed to put a white pawn on f7. However, even if we remove a black pawn, black is missing only one unit (a knight -- correcting an oversight noted by Separatrix).




              6.




              White's check with the bishop on g4 had to be a capture -- yet another capture by White, and this capture, too, is invalid for the same reason as given in 5.




              7.




              White's bishop on g4 and rook on c3 are giving an impossible double check.




              8.




              4. and 6. mean both kings are in check at the same time.




              9.




              White bishop on b1, though it wasn't born there and couldn't have moved there.




              10.




              White has two bishops on white squares but has eight pawns and thus has no promoted pieces.







              share|improve this answer


























              • #4 is incorrect: Black last move could be Ng2-h4.

                – Evargalo
                Jan 4 at 7:11











              • @Evargalo Oh yes, fair point.

                – Rosie F
                Jan 4 at 7:45











              • #5 Black has both rooks, missing a knight.

                – Separatrix
                Jan 4 at 11:53














              7












              7








              7







              Here's my 10 (I didn't look at anyone else's answers, honest)



              1.




              Nine black pawns.




              2.




              Black's h8 rook could not have left rank 8, yet black has two rooks, entailing a promotion from a tenth black pawn!




              3.




              Black pawns have made at least five captures, e.g. cxbxa3 and dxexfxg3, but White is missing only four units (queen, rook and two knights)




              4.




              Black's check with the bishop on f3 had to be a capture -- yet another capture by Black, and this capture, too, is invalid for the same reason as given in 3.




              5.




              White pawns have made at least three captures towards the kingside, e.g. bxcxd5 and dxe3. Moreover, two captures fxexf or fxgxf are needed to put a white pawn on f7. However, even if we remove a black pawn, black is missing only one unit (a knight -- correcting an oversight noted by Separatrix).




              6.




              White's check with the bishop on g4 had to be a capture -- yet another capture by White, and this capture, too, is invalid for the same reason as given in 5.




              7.




              White's bishop on g4 and rook on c3 are giving an impossible double check.




              8.




              4. and 6. mean both kings are in check at the same time.




              9.




              White bishop on b1, though it wasn't born there and couldn't have moved there.




              10.




              White has two bishops on white squares but has eight pawns and thus has no promoted pieces.







              share|improve this answer















              Here's my 10 (I didn't look at anyone else's answers, honest)



              1.




              Nine black pawns.




              2.




              Black's h8 rook could not have left rank 8, yet black has two rooks, entailing a promotion from a tenth black pawn!




              3.




              Black pawns have made at least five captures, e.g. cxbxa3 and dxexfxg3, but White is missing only four units (queen, rook and two knights)




              4.




              Black's check with the bishop on f3 had to be a capture -- yet another capture by Black, and this capture, too, is invalid for the same reason as given in 3.




              5.




              White pawns have made at least three captures towards the kingside, e.g. bxcxd5 and dxe3. Moreover, two captures fxexf or fxgxf are needed to put a white pawn on f7. However, even if we remove a black pawn, black is missing only one unit (a knight -- correcting an oversight noted by Separatrix).




              6.




              White's check with the bishop on g4 had to be a capture -- yet another capture by White, and this capture, too, is invalid for the same reason as given in 5.




              7.




              White's bishop on g4 and rook on c3 are giving an impossible double check.




              8.




              4. and 6. mean both kings are in check at the same time.




              9.




              White bishop on b1, though it wasn't born there and couldn't have moved there.




              10.




              White has two bishops on white squares but has eight pawns and thus has no promoted pieces.








              share|improve this answer














              share|improve this answer



              share|improve this answer








              edited Jan 5 at 8:01

























              answered Jan 3 at 18:59









              Rosie FRosie F

              5,7182943




              5,7182943













              • #4 is incorrect: Black last move could be Ng2-h4.

                – Evargalo
                Jan 4 at 7:11











              • @Evargalo Oh yes, fair point.

                – Rosie F
                Jan 4 at 7:45











              • #5 Black has both rooks, missing a knight.

                – Separatrix
                Jan 4 at 11:53



















              • #4 is incorrect: Black last move could be Ng2-h4.

                – Evargalo
                Jan 4 at 7:11











              • @Evargalo Oh yes, fair point.

                – Rosie F
                Jan 4 at 7:45











              • #5 Black has both rooks, missing a knight.

                – Separatrix
                Jan 4 at 11:53

















              #4 is incorrect: Black last move could be Ng2-h4.

              – Evargalo
              Jan 4 at 7:11





              #4 is incorrect: Black last move could be Ng2-h4.

              – Evargalo
              Jan 4 at 7:11













              @Evargalo Oh yes, fair point.

              – Rosie F
              Jan 4 at 7:45





              @Evargalo Oh yes, fair point.

              – Rosie F
              Jan 4 at 7:45













              #5 Black has both rooks, missing a knight.

              – Separatrix
              Jan 4 at 11:53





              #5 Black has both rooks, missing a knight.

              – Separatrix
              Jan 4 at 11:53











              6














              I'm not sure how you're counting, but here's my answer. Possibly an incomplete answer.



              1.




              Both kings are in check.




              2.




              The black king is in check from two pieces.




              3.




              There are 9 black pawns. Removing the one at g3 makes the pawns at a3 and b2 valid via capturing white officers.




              4.




              There are not enough captured black pieces for the white pawns to be in those positions. The f7 pawn in particular is in a suspicious location, though this pawn configuration is possible if there were enough black officers to capture.




              5.




              The white bishop on b1 is in an impossible location.




              6.




              The white bishops are both on white squares, but all of the pawns are present.




              7.




              The black rook at a1 is in an impossible location... Sort of. It would be possible via promotion, but there already too many black pawns for that to be possible.




              8.




              There should be a black rook in the upper-right corner because there's no way it could get out with the bishop at f8.




              That's all I see. I might be lumping together multiple items by your count into one.






              share|improve this answer




























                6














                I'm not sure how you're counting, but here's my answer. Possibly an incomplete answer.



                1.




                Both kings are in check.




                2.




                The black king is in check from two pieces.




                3.




                There are 9 black pawns. Removing the one at g3 makes the pawns at a3 and b2 valid via capturing white officers.




                4.




                There are not enough captured black pieces for the white pawns to be in those positions. The f7 pawn in particular is in a suspicious location, though this pawn configuration is possible if there were enough black officers to capture.




                5.




                The white bishop on b1 is in an impossible location.




                6.




                The white bishops are both on white squares, but all of the pawns are present.




                7.




                The black rook at a1 is in an impossible location... Sort of. It would be possible via promotion, but there already too many black pawns for that to be possible.




                8.




                There should be a black rook in the upper-right corner because there's no way it could get out with the bishop at f8.




                That's all I see. I might be lumping together multiple items by your count into one.






                share|improve this answer


























                  6












                  6








                  6







                  I'm not sure how you're counting, but here's my answer. Possibly an incomplete answer.



                  1.




                  Both kings are in check.




                  2.




                  The black king is in check from two pieces.




                  3.




                  There are 9 black pawns. Removing the one at g3 makes the pawns at a3 and b2 valid via capturing white officers.




                  4.




                  There are not enough captured black pieces for the white pawns to be in those positions. The f7 pawn in particular is in a suspicious location, though this pawn configuration is possible if there were enough black officers to capture.




                  5.




                  The white bishop on b1 is in an impossible location.




                  6.




                  The white bishops are both on white squares, but all of the pawns are present.




                  7.




                  The black rook at a1 is in an impossible location... Sort of. It would be possible via promotion, but there already too many black pawns for that to be possible.




                  8.




                  There should be a black rook in the upper-right corner because there's no way it could get out with the bishop at f8.




                  That's all I see. I might be lumping together multiple items by your count into one.






                  share|improve this answer













                  I'm not sure how you're counting, but here's my answer. Possibly an incomplete answer.



                  1.




                  Both kings are in check.




                  2.




                  The black king is in check from two pieces.




                  3.




                  There are 9 black pawns. Removing the one at g3 makes the pawns at a3 and b2 valid via capturing white officers.




                  4.




                  There are not enough captured black pieces for the white pawns to be in those positions. The f7 pawn in particular is in a suspicious location, though this pawn configuration is possible if there were enough black officers to capture.




                  5.




                  The white bishop on b1 is in an impossible location.




                  6.




                  The white bishops are both on white squares, but all of the pawns are present.




                  7.




                  The black rook at a1 is in an impossible location... Sort of. It would be possible via promotion, but there already too many black pawns for that to be possible.




                  8.




                  There should be a black rook in the upper-right corner because there's no way it could get out with the bishop at f8.




                  That's all I see. I might be lumping together multiple items by your count into one.







                  share|improve this answer












                  share|improve this answer



                  share|improve this answer










                  answered Jan 3 at 19:48









                  BeefsterBeefster

                  1612




                  1612























                      5















                      1. bishop in g4 in check

                      2. rook in c3 in check

                      3. bishop in f3 in check

                      4. rook in a1 cant get there

                      5. 9 black pawns

                      6. pawn in a3 cant get there

                      7. pawn in g3 cant get there

                      8. pawn in f7 cant get there

                      9. 2 white bishop on white squares

                      10. pawn in e3 cant get there

                      11. bishop in b1 cant get there







                      share|improve this answer





















                      • 1





                        Your #4 and #12 are the same. The A3 pawn could have come from the C column by capturing twice, same goes for G3 from the D column, although since black is only missing 2 pieces, one of these must be illegal.

                        – Nuclear Wang
                        Jan 3 at 15:36
















                      5















                      1. bishop in g4 in check

                      2. rook in c3 in check

                      3. bishop in f3 in check

                      4. rook in a1 cant get there

                      5. 9 black pawns

                      6. pawn in a3 cant get there

                      7. pawn in g3 cant get there

                      8. pawn in f7 cant get there

                      9. 2 white bishop on white squares

                      10. pawn in e3 cant get there

                      11. bishop in b1 cant get there







                      share|improve this answer





















                      • 1





                        Your #4 and #12 are the same. The A3 pawn could have come from the C column by capturing twice, same goes for G3 from the D column, although since black is only missing 2 pieces, one of these must be illegal.

                        – Nuclear Wang
                        Jan 3 at 15:36














                      5












                      5








                      5








                      1. bishop in g4 in check

                      2. rook in c3 in check

                      3. bishop in f3 in check

                      4. rook in a1 cant get there

                      5. 9 black pawns

                      6. pawn in a3 cant get there

                      7. pawn in g3 cant get there

                      8. pawn in f7 cant get there

                      9. 2 white bishop on white squares

                      10. pawn in e3 cant get there

                      11. bishop in b1 cant get there







                      share|improve this answer
















                      1. bishop in g4 in check

                      2. rook in c3 in check

                      3. bishop in f3 in check

                      4. rook in a1 cant get there

                      5. 9 black pawns

                      6. pawn in a3 cant get there

                      7. pawn in g3 cant get there

                      8. pawn in f7 cant get there

                      9. 2 white bishop on white squares

                      10. pawn in e3 cant get there

                      11. bishop in b1 cant get there








                      share|improve this answer














                      share|improve this answer



                      share|improve this answer








                      edited Jan 3 at 15:38

























                      answered Jan 3 at 15:16









                      piratepirate

                      661118




                      661118








                      • 1





                        Your #4 and #12 are the same. The A3 pawn could have come from the C column by capturing twice, same goes for G3 from the D column, although since black is only missing 2 pieces, one of these must be illegal.

                        – Nuclear Wang
                        Jan 3 at 15:36














                      • 1





                        Your #4 and #12 are the same. The A3 pawn could have come from the C column by capturing twice, same goes for G3 from the D column, although since black is only missing 2 pieces, one of these must be illegal.

                        – Nuclear Wang
                        Jan 3 at 15:36








                      1




                      1





                      Your #4 and #12 are the same. The A3 pawn could have come from the C column by capturing twice, same goes for G3 from the D column, although since black is only missing 2 pieces, one of these must be illegal.

                      – Nuclear Wang
                      Jan 3 at 15:36





                      Your #4 and #12 are the same. The A3 pawn could have come from the C column by capturing twice, same goes for G3 from the D column, although since black is only missing 2 pieces, one of these must be illegal.

                      – Nuclear Wang
                      Jan 3 at 15:36











                      4














                      1.




                      Black has 9 pawns.




                      2.




                      Both kings are in check.




                      3.




                      The black king is in double check. This can only happen if one of the checking pieces shielded the opponent king from the other prior to the check. This is not possible with the took on c3 and the bishop on h4.




                      4.




                      The white bishop on b1 could never have reached this field with the white pawns on a2, c2.




                      5.




                      White has two bishops on white fields. This can only happen after a pawn has been promoted. White however still has their complete set of 8 pawns.




                      6.




                      With white pawns on the e and g files and a black pawn on f6, the white pawn on f7 can reach this field only with at least 2 captures. However, black has lost only 1 piece and all pawns (in fact even one surplus pawn, he. item 1), thus never had a promoted piece.




                      7.




                      White's pawn structure implies at least 5 captures (bxc, cxd; dxe; fxe, exf) but black has lost a single piece only and no promotion so far (cf. item 6)




                      8.




                      With black pawns on e7,f6,g7,h7 and a black bishop on f8, the black rook initially on h8 could only have reached the field g8.




                      9.




                      The black king is in check so white moved last. Since the white bishop on h4 can only move along the diagonal to c8 the black king must have been in check before the move; or black put itself into check by illegally moving a pinned piece; or white moved a different piece to open the diagonal, but white does not have a piece that could have moved this way [not sure if this counts though as the checking situation is impossible in the first place, cf. items 2 & 3]




                      10.




                      Black is in an 'impossible' double check and puts white in check (cf. items 2 & 3). That can only happen if black doesn't move out of a check, thus playing illegally.







                      share|improve this answer






























                        4














                        1.




                        Black has 9 pawns.




                        2.




                        Both kings are in check.




                        3.




                        The black king is in double check. This can only happen if one of the checking pieces shielded the opponent king from the other prior to the check. This is not possible with the took on c3 and the bishop on h4.




                        4.




                        The white bishop on b1 could never have reached this field with the white pawns on a2, c2.




                        5.




                        White has two bishops on white fields. This can only happen after a pawn has been promoted. White however still has their complete set of 8 pawns.




                        6.




                        With white pawns on the e and g files and a black pawn on f6, the white pawn on f7 can reach this field only with at least 2 captures. However, black has lost only 1 piece and all pawns (in fact even one surplus pawn, he. item 1), thus never had a promoted piece.




                        7.




                        White's pawn structure implies at least 5 captures (bxc, cxd; dxe; fxe, exf) but black has lost a single piece only and no promotion so far (cf. item 6)




                        8.




                        With black pawns on e7,f6,g7,h7 and a black bishop on f8, the black rook initially on h8 could only have reached the field g8.




                        9.




                        The black king is in check so white moved last. Since the white bishop on h4 can only move along the diagonal to c8 the black king must have been in check before the move; or black put itself into check by illegally moving a pinned piece; or white moved a different piece to open the diagonal, but white does not have a piece that could have moved this way [not sure if this counts though as the checking situation is impossible in the first place, cf. items 2 & 3]




                        10.




                        Black is in an 'impossible' double check and puts white in check (cf. items 2 & 3). That can only happen if black doesn't move out of a check, thus playing illegally.







                        share|improve this answer




























                          4












                          4








                          4







                          1.




                          Black has 9 pawns.




                          2.




                          Both kings are in check.




                          3.




                          The black king is in double check. This can only happen if one of the checking pieces shielded the opponent king from the other prior to the check. This is not possible with the took on c3 and the bishop on h4.




                          4.




                          The white bishop on b1 could never have reached this field with the white pawns on a2, c2.




                          5.




                          White has two bishops on white fields. This can only happen after a pawn has been promoted. White however still has their complete set of 8 pawns.




                          6.




                          With white pawns on the e and g files and a black pawn on f6, the white pawn on f7 can reach this field only with at least 2 captures. However, black has lost only 1 piece and all pawns (in fact even one surplus pawn, he. item 1), thus never had a promoted piece.




                          7.




                          White's pawn structure implies at least 5 captures (bxc, cxd; dxe; fxe, exf) but black has lost a single piece only and no promotion so far (cf. item 6)




                          8.




                          With black pawns on e7,f6,g7,h7 and a black bishop on f8, the black rook initially on h8 could only have reached the field g8.




                          9.




                          The black king is in check so white moved last. Since the white bishop on h4 can only move along the diagonal to c8 the black king must have been in check before the move; or black put itself into check by illegally moving a pinned piece; or white moved a different piece to open the diagonal, but white does not have a piece that could have moved this way [not sure if this counts though as the checking situation is impossible in the first place, cf. items 2 & 3]




                          10.




                          Black is in an 'impossible' double check and puts white in check (cf. items 2 & 3). That can only happen if black doesn't move out of a check, thus playing illegally.







                          share|improve this answer















                          1.




                          Black has 9 pawns.




                          2.




                          Both kings are in check.




                          3.




                          The black king is in double check. This can only happen if one of the checking pieces shielded the opponent king from the other prior to the check. This is not possible with the took on c3 and the bishop on h4.




                          4.




                          The white bishop on b1 could never have reached this field with the white pawns on a2, c2.




                          5.




                          White has two bishops on white fields. This can only happen after a pawn has been promoted. White however still has their complete set of 8 pawns.




                          6.




                          With white pawns on the e and g files and a black pawn on f6, the white pawn on f7 can reach this field only with at least 2 captures. However, black has lost only 1 piece and all pawns (in fact even one surplus pawn, he. item 1), thus never had a promoted piece.




                          7.




                          White's pawn structure implies at least 5 captures (bxc, cxd; dxe; fxe, exf) but black has lost a single piece only and no promotion so far (cf. item 6)




                          8.




                          With black pawns on e7,f6,g7,h7 and a black bishop on f8, the black rook initially on h8 could only have reached the field g8.




                          9.




                          The black king is in check so white moved last. Since the white bishop on h4 can only move along the diagonal to c8 the black king must have been in check before the move; or black put itself into check by illegally moving a pinned piece; or white moved a different piece to open the diagonal, but white does not have a piece that could have moved this way [not sure if this counts though as the checking situation is impossible in the first place, cf. items 2 & 3]




                          10.




                          Black is in an 'impossible' double check and puts white in check (cf. items 2 & 3). That can only happen if black doesn't move out of a check, thus playing illegally.








                          share|improve this answer














                          share|improve this answer



                          share|improve this answer








                          edited Jan 7 at 11:36









                          Pimgd

                          1033




                          1033










                          answered Jan 4 at 1:47









                          collapsarcollapsar

                          20714




                          20714























                              3














                              I only managed to find 10 so far, here it is:




                              1. White bishop in B1. It's not possible in a normal game.

                              2. Two white bishops in the white square, while still having 8 pawns.

                              3. Black Rook in A1. It's not possible in a normal game.

                              4. Black Rook going out from H8 while Bishop hasn't moved from F8.

                              5. White Rook missing from A1. Stolen and replaced by black rook? :P

                              6. Black pawn totaling 9. It was supposed to be only 8 in a game.

                              7. White king under check, while black king also in check.

                              8. Black king under two different check, from Rook in C3 and Bishop on G4

                              9. White pawn structure. It just simply weird to have a pawn on F7, D5 while also having a pawn on E3, while black only missing a knight.

                              10. Both kings is under check, is simply impossible.







                              share|improve this answer
























                              • Black rook in A1 is possible (in isolation) as a promoted pawn having captured the White rook normally there. #7 and #10 are duplicates.

                                – Separatrix
                                Jan 4 at 12:06








                              • 1





                                @Separatrix: Black has 9 pawns (in itself illegal), however, so there has not been a promotion. Or, if there was, black has been playing with at least 10 pawns at some moment in time, adding another illegal pawn (unless surplus pawns count as a single rule violation w/o regard to their number).

                                – collapsar
                                Jan 4 at 17:25
















                              3














                              I only managed to find 10 so far, here it is:




                              1. White bishop in B1. It's not possible in a normal game.

                              2. Two white bishops in the white square, while still having 8 pawns.

                              3. Black Rook in A1. It's not possible in a normal game.

                              4. Black Rook going out from H8 while Bishop hasn't moved from F8.

                              5. White Rook missing from A1. Stolen and replaced by black rook? :P

                              6. Black pawn totaling 9. It was supposed to be only 8 in a game.

                              7. White king under check, while black king also in check.

                              8. Black king under two different check, from Rook in C3 and Bishop on G4

                              9. White pawn structure. It just simply weird to have a pawn on F7, D5 while also having a pawn on E3, while black only missing a knight.

                              10. Both kings is under check, is simply impossible.







                              share|improve this answer
























                              • Black rook in A1 is possible (in isolation) as a promoted pawn having captured the White rook normally there. #7 and #10 are duplicates.

                                – Separatrix
                                Jan 4 at 12:06








                              • 1





                                @Separatrix: Black has 9 pawns (in itself illegal), however, so there has not been a promotion. Or, if there was, black has been playing with at least 10 pawns at some moment in time, adding another illegal pawn (unless surplus pawns count as a single rule violation w/o regard to their number).

                                – collapsar
                                Jan 4 at 17:25














                              3












                              3








                              3







                              I only managed to find 10 so far, here it is:




                              1. White bishop in B1. It's not possible in a normal game.

                              2. Two white bishops in the white square, while still having 8 pawns.

                              3. Black Rook in A1. It's not possible in a normal game.

                              4. Black Rook going out from H8 while Bishop hasn't moved from F8.

                              5. White Rook missing from A1. Stolen and replaced by black rook? :P

                              6. Black pawn totaling 9. It was supposed to be only 8 in a game.

                              7. White king under check, while black king also in check.

                              8. Black king under two different check, from Rook in C3 and Bishop on G4

                              9. White pawn structure. It just simply weird to have a pawn on F7, D5 while also having a pawn on E3, while black only missing a knight.

                              10. Both kings is under check, is simply impossible.







                              share|improve this answer













                              I only managed to find 10 so far, here it is:




                              1. White bishop in B1. It's not possible in a normal game.

                              2. Two white bishops in the white square, while still having 8 pawns.

                              3. Black Rook in A1. It's not possible in a normal game.

                              4. Black Rook going out from H8 while Bishop hasn't moved from F8.

                              5. White Rook missing from A1. Stolen and replaced by black rook? :P

                              6. Black pawn totaling 9. It was supposed to be only 8 in a game.

                              7. White king under check, while black king also in check.

                              8. Black king under two different check, from Rook in C3 and Bishop on G4

                              9. White pawn structure. It just simply weird to have a pawn on F7, D5 while also having a pawn on E3, while black only missing a knight.

                              10. Both kings is under check, is simply impossible.








                              share|improve this answer












                              share|improve this answer



                              share|improve this answer










                              answered Jan 4 at 1:27









                              MukyuuMukyuu

                              300112




                              300112













                              • Black rook in A1 is possible (in isolation) as a promoted pawn having captured the White rook normally there. #7 and #10 are duplicates.

                                – Separatrix
                                Jan 4 at 12:06








                              • 1





                                @Separatrix: Black has 9 pawns (in itself illegal), however, so there has not been a promotion. Or, if there was, black has been playing with at least 10 pawns at some moment in time, adding another illegal pawn (unless surplus pawns count as a single rule violation w/o regard to their number).

                                – collapsar
                                Jan 4 at 17:25



















                              • Black rook in A1 is possible (in isolation) as a promoted pawn having captured the White rook normally there. #7 and #10 are duplicates.

                                – Separatrix
                                Jan 4 at 12:06








                              • 1





                                @Separatrix: Black has 9 pawns (in itself illegal), however, so there has not been a promotion. Or, if there was, black has been playing with at least 10 pawns at some moment in time, adding another illegal pawn (unless surplus pawns count as a single rule violation w/o regard to their number).

                                – collapsar
                                Jan 4 at 17:25

















                              Black rook in A1 is possible (in isolation) as a promoted pawn having captured the White rook normally there. #7 and #10 are duplicates.

                              – Separatrix
                              Jan 4 at 12:06







                              Black rook in A1 is possible (in isolation) as a promoted pawn having captured the White rook normally there. #7 and #10 are duplicates.

                              – Separatrix
                              Jan 4 at 12:06






                              1




                              1





                              @Separatrix: Black has 9 pawns (in itself illegal), however, so there has not been a promotion. Or, if there was, black has been playing with at least 10 pawns at some moment in time, adding another illegal pawn (unless surplus pawns count as a single rule violation w/o regard to their number).

                              – collapsar
                              Jan 4 at 17:25





                              @Separatrix: Black has 9 pawns (in itself illegal), however, so there has not been a promotion. Or, if there was, black has been playing with at least 10 pawns at some moment in time, adding another illegal pawn (unless surplus pawns count as a single rule violation w/o regard to their number).

                              – collapsar
                              Jan 4 at 17:25











                              3














                              Definitely illegal



                              1.




                              Black has 9 pawns




                              2.




                              Rook a1 could be legal in isolation as a promoted pawn, but in light of #1, I'm declaring it illegal




                              3.




                              Bishop b1 cannot get there




                              4.




                              Both white bishops are on white, white has 8 pawns, one bishop is illegal




                              5.




                              Both kings are in check




                              6.




                              Black king is in illegal double check, could not be discovered by legal movement of either offending piece




                              7.




                              Black rook starting h8 could not have legally escaped, due to number of remaining black pawns, absence of rook considered illegal




                              8.




                              White pawns are at least 3 places off home columns, black still has 16 pieces, though missing a knight, at least two illegal places off home




                              9.




                              Pawn f7 is illegal, cannot have captured the required 2 pieces to return to home column behind black pawn as black not missing enough pieces




                              10.




                              Even assuming pawn g3 to be the interloper, black pawns are a minimum of 4 places off home column, white is missing 4 pieces, but rook a1 could only have been captured by a knight or queen, at least one illegal place off home




                              Not illegal but might look it



                              1.




                              Bishop f3 check could be discovered by movement of knight h4 from g2, otherwise it's really hard to get into that position





                              1. This ties into illegal #10



                              Assuming bishop b1 to have been placed there at the start of the game, white rook starting a1 could not have legally escaped, but could have been captured by a knight or queen. Black bishop could not have been used.







                              share|improve this answer






























                                3














                                Definitely illegal



                                1.




                                Black has 9 pawns




                                2.




                                Rook a1 could be legal in isolation as a promoted pawn, but in light of #1, I'm declaring it illegal




                                3.




                                Bishop b1 cannot get there




                                4.




                                Both white bishops are on white, white has 8 pawns, one bishop is illegal




                                5.




                                Both kings are in check




                                6.




                                Black king is in illegal double check, could not be discovered by legal movement of either offending piece




                                7.




                                Black rook starting h8 could not have legally escaped, due to number of remaining black pawns, absence of rook considered illegal




                                8.




                                White pawns are at least 3 places off home columns, black still has 16 pieces, though missing a knight, at least two illegal places off home




                                9.




                                Pawn f7 is illegal, cannot have captured the required 2 pieces to return to home column behind black pawn as black not missing enough pieces




                                10.




                                Even assuming pawn g3 to be the interloper, black pawns are a minimum of 4 places off home column, white is missing 4 pieces, but rook a1 could only have been captured by a knight or queen, at least one illegal place off home




                                Not illegal but might look it



                                1.




                                Bishop f3 check could be discovered by movement of knight h4 from g2, otherwise it's really hard to get into that position





                                1. This ties into illegal #10



                                Assuming bishop b1 to have been placed there at the start of the game, white rook starting a1 could not have legally escaped, but could have been captured by a knight or queen. Black bishop could not have been used.







                                share|improve this answer




























                                  3












                                  3








                                  3







                                  Definitely illegal



                                  1.




                                  Black has 9 pawns




                                  2.




                                  Rook a1 could be legal in isolation as a promoted pawn, but in light of #1, I'm declaring it illegal




                                  3.




                                  Bishop b1 cannot get there




                                  4.




                                  Both white bishops are on white, white has 8 pawns, one bishop is illegal




                                  5.




                                  Both kings are in check




                                  6.




                                  Black king is in illegal double check, could not be discovered by legal movement of either offending piece




                                  7.




                                  Black rook starting h8 could not have legally escaped, due to number of remaining black pawns, absence of rook considered illegal




                                  8.




                                  White pawns are at least 3 places off home columns, black still has 16 pieces, though missing a knight, at least two illegal places off home




                                  9.




                                  Pawn f7 is illegal, cannot have captured the required 2 pieces to return to home column behind black pawn as black not missing enough pieces




                                  10.




                                  Even assuming pawn g3 to be the interloper, black pawns are a minimum of 4 places off home column, white is missing 4 pieces, but rook a1 could only have been captured by a knight or queen, at least one illegal place off home




                                  Not illegal but might look it



                                  1.




                                  Bishop f3 check could be discovered by movement of knight h4 from g2, otherwise it's really hard to get into that position





                                  1. This ties into illegal #10



                                  Assuming bishop b1 to have been placed there at the start of the game, white rook starting a1 could not have legally escaped, but could have been captured by a knight or queen. Black bishop could not have been used.







                                  share|improve this answer















                                  Definitely illegal



                                  1.




                                  Black has 9 pawns




                                  2.




                                  Rook a1 could be legal in isolation as a promoted pawn, but in light of #1, I'm declaring it illegal




                                  3.




                                  Bishop b1 cannot get there




                                  4.




                                  Both white bishops are on white, white has 8 pawns, one bishop is illegal




                                  5.




                                  Both kings are in check




                                  6.




                                  Black king is in illegal double check, could not be discovered by legal movement of either offending piece




                                  7.




                                  Black rook starting h8 could not have legally escaped, due to number of remaining black pawns, absence of rook considered illegal




                                  8.




                                  White pawns are at least 3 places off home columns, black still has 16 pieces, though missing a knight, at least two illegal places off home




                                  9.




                                  Pawn f7 is illegal, cannot have captured the required 2 pieces to return to home column behind black pawn as black not missing enough pieces




                                  10.




                                  Even assuming pawn g3 to be the interloper, black pawns are a minimum of 4 places off home column, white is missing 4 pieces, but rook a1 could only have been captured by a knight or queen, at least one illegal place off home




                                  Not illegal but might look it



                                  1.




                                  Bishop f3 check could be discovered by movement of knight h4 from g2, otherwise it's really hard to get into that position





                                  1. This ties into illegal #10



                                  Assuming bishop b1 to have been placed there at the start of the game, white rook starting a1 could not have legally escaped, but could have been captured by a knight or queen. Black bishop could not have been used.








                                  share|improve this answer














                                  share|improve this answer



                                  share|improve this answer








                                  edited Jan 4 at 12:38

























                                  answered Jan 4 at 12:32









                                  SeparatrixSeparatrix

                                  27116




                                  27116























                                      2
















                                      • Bb1, no legal way to reach there



                                      • -


                                      - Ra1, no legal way to reach there, due to Bb1






                                      • Kh1, in check, other King in check



                                      • -


                                      Kc8, in double check, other King in check






                                      • Kc8, in an impossible double check, no legal way to give that double check



                                      • -


                                      f7, no legal way to reach there






                                      • Bf8, no legal way to reach there



                                      • -


                                      2 white bishops, with 8 pawns. Does this count as two illegals?







                                      share|improve this answer






























                                        2
















                                        • Bb1, no legal way to reach there



                                        • -


                                        - Ra1, no legal way to reach there, due to Bb1






                                        • Kh1, in check, other King in check



                                        • -


                                        Kc8, in double check, other King in check






                                        • Kc8, in an impossible double check, no legal way to give that double check



                                        • -


                                        f7, no legal way to reach there






                                        • Bf8, no legal way to reach there



                                        • -


                                        2 white bishops, with 8 pawns. Does this count as two illegals?







                                        share|improve this answer




























                                          2












                                          2








                                          2









                                          • Bb1, no legal way to reach there



                                          • -


                                          - Ra1, no legal way to reach there, due to Bb1






                                          • Kh1, in check, other King in check



                                          • -


                                          Kc8, in double check, other King in check






                                          • Kc8, in an impossible double check, no legal way to give that double check



                                          • -


                                          f7, no legal way to reach there






                                          • Bf8, no legal way to reach there



                                          • -


                                          2 white bishops, with 8 pawns. Does this count as two illegals?







                                          share|improve this answer

















                                          • Bb1, no legal way to reach there



                                          • -


                                          - Ra1, no legal way to reach there, due to Bb1






                                          • Kh1, in check, other King in check



                                          • -


                                          Kc8, in double check, other King in check






                                          • Kc8, in an impossible double check, no legal way to give that double check



                                          • -


                                          f7, no legal way to reach there






                                          • Bf8, no legal way to reach there



                                          • -


                                          2 white bishops, with 8 pawns. Does this count as two illegals?








                                          share|improve this answer














                                          share|improve this answer



                                          share|improve this answer








                                          edited Jan 4 at 23:57

























                                          answered Jan 4 at 16:22









                                          KashyapKashyap

                                          1214




                                          1214























                                              1














                                              1.




                                              Both kings are in check.




                                              2.




                                              Black is in check by 2 pieces, which, as arranged, could not have co-participated in a discovered check.




                                              3.




                                              Black has 9 pawns.




                                              4.




                                              White's bishop could not reach B1 without a pawn moving backwards.




                                              5.




                                              Black's rook could not have reached A1.




                                              6.




                                              Both White bishops are on the same color.




                                              7.




                                              White pawn at F7 cannot have passed through Black pawn at F6, nor could it have reached that position through capture as Black is missing at most one piece.




                                              8.




                                              White pawn at position E3 is impossible without capturing more material than Black lost.




                                              9.




                                              White pawn from file B is unaccounted for. With Black down at most one piece, it should be in file A, B, or C.




                                              10.




                                              Black rook from H8 could not have left starting position (other than G8 and it's not there either).




                                              A.




                                              It's tempting to say that positioning of Black's pawns is illegal, but I've already taken credit for one illegal pawn. If A3 is that illegal pawn, then it could have started at, say, A6. Then B2 could be reached by capture of 1 piece, and G3 could be reached by capture of 3 pieces, and White is down in material by 4 pieces, so it's not impossible. It depends on whether Black started out with 16 pieces within its two starting ranks (substituting a pawn for a knight), or if the 9th pawn started the game outside of those two ranks.







                                              share|improve this answer




























                                                1














                                                1.




                                                Both kings are in check.




                                                2.




                                                Black is in check by 2 pieces, which, as arranged, could not have co-participated in a discovered check.




                                                3.




                                                Black has 9 pawns.




                                                4.




                                                White's bishop could not reach B1 without a pawn moving backwards.




                                                5.




                                                Black's rook could not have reached A1.




                                                6.




                                                Both White bishops are on the same color.




                                                7.




                                                White pawn at F7 cannot have passed through Black pawn at F6, nor could it have reached that position through capture as Black is missing at most one piece.




                                                8.




                                                White pawn at position E3 is impossible without capturing more material than Black lost.




                                                9.




                                                White pawn from file B is unaccounted for. With Black down at most one piece, it should be in file A, B, or C.




                                                10.




                                                Black rook from H8 could not have left starting position (other than G8 and it's not there either).




                                                A.




                                                It's tempting to say that positioning of Black's pawns is illegal, but I've already taken credit for one illegal pawn. If A3 is that illegal pawn, then it could have started at, say, A6. Then B2 could be reached by capture of 1 piece, and G3 could be reached by capture of 3 pieces, and White is down in material by 4 pieces, so it's not impossible. It depends on whether Black started out with 16 pieces within its two starting ranks (substituting a pawn for a knight), or if the 9th pawn started the game outside of those two ranks.







                                                share|improve this answer


























                                                  1












                                                  1








                                                  1







                                                  1.




                                                  Both kings are in check.




                                                  2.




                                                  Black is in check by 2 pieces, which, as arranged, could not have co-participated in a discovered check.




                                                  3.




                                                  Black has 9 pawns.




                                                  4.




                                                  White's bishop could not reach B1 without a pawn moving backwards.




                                                  5.




                                                  Black's rook could not have reached A1.




                                                  6.




                                                  Both White bishops are on the same color.




                                                  7.




                                                  White pawn at F7 cannot have passed through Black pawn at F6, nor could it have reached that position through capture as Black is missing at most one piece.




                                                  8.




                                                  White pawn at position E3 is impossible without capturing more material than Black lost.




                                                  9.




                                                  White pawn from file B is unaccounted for. With Black down at most one piece, it should be in file A, B, or C.




                                                  10.




                                                  Black rook from H8 could not have left starting position (other than G8 and it's not there either).




                                                  A.




                                                  It's tempting to say that positioning of Black's pawns is illegal, but I've already taken credit for one illegal pawn. If A3 is that illegal pawn, then it could have started at, say, A6. Then B2 could be reached by capture of 1 piece, and G3 could be reached by capture of 3 pieces, and White is down in material by 4 pieces, so it's not impossible. It depends on whether Black started out with 16 pieces within its two starting ranks (substituting a pawn for a knight), or if the 9th pawn started the game outside of those two ranks.







                                                  share|improve this answer













                                                  1.




                                                  Both kings are in check.




                                                  2.




                                                  Black is in check by 2 pieces, which, as arranged, could not have co-participated in a discovered check.




                                                  3.




                                                  Black has 9 pawns.




                                                  4.




                                                  White's bishop could not reach B1 without a pawn moving backwards.




                                                  5.




                                                  Black's rook could not have reached A1.




                                                  6.




                                                  Both White bishops are on the same color.




                                                  7.




                                                  White pawn at F7 cannot have passed through Black pawn at F6, nor could it have reached that position through capture as Black is missing at most one piece.




                                                  8.




                                                  White pawn at position E3 is impossible without capturing more material than Black lost.




                                                  9.




                                                  White pawn from file B is unaccounted for. With Black down at most one piece, it should be in file A, B, or C.




                                                  10.




                                                  Black rook from H8 could not have left starting position (other than G8 and it's not there either).




                                                  A.




                                                  It's tempting to say that positioning of Black's pawns is illegal, but I've already taken credit for one illegal pawn. If A3 is that illegal pawn, then it could have started at, say, A6. Then B2 could be reached by capture of 1 piece, and G3 could be reached by capture of 3 pieces, and White is down in material by 4 pieces, so it's not impossible. It depends on whether Black started out with 16 pieces within its two starting ranks (substituting a pawn for a knight), or if the 9th pawn started the game outside of those two ranks.








                                                  share|improve this answer












                                                  share|improve this answer



                                                  share|improve this answer










                                                  answered Jan 5 at 16:10









                                                  phatfingersphatfingers

                                                  1111




                                                  1111























                                                      0














                                                      1.




                                                      Black has a 9th pawn.




                                                      2.




                                                      Both White and Black are in check simultaneously.




                                                      3.




                                                      Black is in double check with the c3 rook and g4 bishop.




                                                      4.




                                                      White's two bishops are on the same color of square. White still has 8 pawns, so one bishop could not have come from promotion. Either a bishop started on the wrong square, or White started with additional pawns and/or bishops.




                                                      5.




                                                      White's bishop on b1, if it started on f1, could not have got to b1 when the diagonal pawns are still in place.




                                                      6.




                                                      Black's rook on a1 could not have got there behind the b1 bishop, if the bishop started there.




                                                      7.




                                                      Black's rook that would have started on h8 could not have gotten out of the g8-h8 box.




                                                      8.




                                                      White's pawns on d5 and e3 and f7 require that at least 5 diagonal movements - captures of Black's pieces - have taken place, but Black is down at most 1 piece (a knight), or started with several additional pieces.




                                                      Fun! But I count only eight.






                                                      share|improve this answer






























                                                        0














                                                        1.




                                                        Black has a 9th pawn.




                                                        2.




                                                        Both White and Black are in check simultaneously.




                                                        3.




                                                        Black is in double check with the c3 rook and g4 bishop.




                                                        4.




                                                        White's two bishops are on the same color of square. White still has 8 pawns, so one bishop could not have come from promotion. Either a bishop started on the wrong square, or White started with additional pawns and/or bishops.




                                                        5.




                                                        White's bishop on b1, if it started on f1, could not have got to b1 when the diagonal pawns are still in place.




                                                        6.




                                                        Black's rook on a1 could not have got there behind the b1 bishop, if the bishop started there.




                                                        7.




                                                        Black's rook that would have started on h8 could not have gotten out of the g8-h8 box.




                                                        8.




                                                        White's pawns on d5 and e3 and f7 require that at least 5 diagonal movements - captures of Black's pieces - have taken place, but Black is down at most 1 piece (a knight), or started with several additional pieces.




                                                        Fun! But I count only eight.






                                                        share|improve this answer




























                                                          0












                                                          0








                                                          0







                                                          1.




                                                          Black has a 9th pawn.




                                                          2.




                                                          Both White and Black are in check simultaneously.




                                                          3.




                                                          Black is in double check with the c3 rook and g4 bishop.




                                                          4.




                                                          White's two bishops are on the same color of square. White still has 8 pawns, so one bishop could not have come from promotion. Either a bishop started on the wrong square, or White started with additional pawns and/or bishops.




                                                          5.




                                                          White's bishop on b1, if it started on f1, could not have got to b1 when the diagonal pawns are still in place.




                                                          6.




                                                          Black's rook on a1 could not have got there behind the b1 bishop, if the bishop started there.




                                                          7.




                                                          Black's rook that would have started on h8 could not have gotten out of the g8-h8 box.




                                                          8.




                                                          White's pawns on d5 and e3 and f7 require that at least 5 diagonal movements - captures of Black's pieces - have taken place, but Black is down at most 1 piece (a knight), or started with several additional pieces.




                                                          Fun! But I count only eight.






                                                          share|improve this answer















                                                          1.




                                                          Black has a 9th pawn.




                                                          2.




                                                          Both White and Black are in check simultaneously.




                                                          3.




                                                          Black is in double check with the c3 rook and g4 bishop.




                                                          4.




                                                          White's two bishops are on the same color of square. White still has 8 pawns, so one bishop could not have come from promotion. Either a bishop started on the wrong square, or White started with additional pawns and/or bishops.




                                                          5.




                                                          White's bishop on b1, if it started on f1, could not have got to b1 when the diagonal pawns are still in place.




                                                          6.




                                                          Black's rook on a1 could not have got there behind the b1 bishop, if the bishop started there.




                                                          7.




                                                          Black's rook that would have started on h8 could not have gotten out of the g8-h8 box.




                                                          8.




                                                          White's pawns on d5 and e3 and f7 require that at least 5 diagonal movements - captures of Black's pieces - have taken place, but Black is down at most 1 piece (a knight), or started with several additional pieces.




                                                          Fun! But I count only eight.







                                                          share|improve this answer














                                                          share|improve this answer



                                                          share|improve this answer








                                                          edited Jan 6 at 3:13

























                                                          answered Jan 6 at 3:00









                                                          BoannBoann

                                                          1014




                                                          1014






























                                                              draft saved

                                                              draft discarded




















































                                                              Thanks for contributing an answer to Puzzling Stack Exchange!


                                                              • Please be sure to answer the question. Provide details and share your research!

                                                              But avoid



                                                              • Asking for help, clarification, or responding to other answers.

                                                              • Making statements based on opinion; back them up with references or personal experience.


                                                              Use MathJax to format equations. MathJax reference.


                                                              To learn more, see our tips on writing great answers.




                                                              draft saved


                                                              draft discarded














                                                              StackExchange.ready(
                                                              function () {
                                                              StackExchange.openid.initPostLogin('.new-post-login', 'https%3a%2f%2fpuzzling.stackexchange.com%2fquestions%2f78081%2fa-very-illegal-position%23new-answer', 'question_page');
                                                              }
                                                              );

                                                              Post as a guest















                                                              Required, but never shown





















































                                                              Required, but never shown














                                                              Required, but never shown












                                                              Required, but never shown







                                                              Required, but never shown

































                                                              Required, but never shown














                                                              Required, but never shown












                                                              Required, but never shown







                                                              Required, but never shown







                                                              Popular posts from this blog

                                                              Human spaceflight

                                                              Can not write log (Is /dev/pts mounted?) - openpty in Ubuntu-on-Windows?

                                                              File:DeusFollowingSea.jpg